2014 dxdy logo

Научный форум dxdy

Математика, Физика, Computer Science, Machine Learning, LaTeX, Механика и Техника, Химия,
Биология и Медицина, Экономика и Финансовая Математика, Гуманитарные науки




Начать новую тему Ответить на тему
 
 две 1000-долларовые задачи от Рона Грэхема
Сообщение16.02.2007, 03:26 
Модератор
Аватара пользователя


11/01/06
5660
Сегодня общался с Роном Грэхемом (один из авторов "Конкретной Математики"), он предложил по $1000 за решение каждой из следующих задачек:

1) Доказать, что найдется бесконечно много натуральных чисел $m$ таких, что
$$\mathop{\text{НОД}}({2m\choose m}, 3\cdot 5\cdot 7) = 1.$$

2) Доказать, что найдется лишь конечное количество натуральных чисел $m$ таких, что
$$\mathop{\text{НОД}}({2m\choose m}, 3\cdot 5\cdot 7\cdot 11) = 1.$$

Добавлено спустя 2 часа 35 минут 57 секунд:

Замечание 1. Биномиальный коэффициент ${2m\choose m}$ не делится на простое число $p$ тогда и только тогда, когда все цифры в $p$-ичном представлении $m$ не превосходят $\frac{p}{2}.$

Замечание 2. Может оказаться полезной эта статья: On the Prime Factor of ${2n \choose n}$, Math. Comp. 29 (1975), 83-92.

 Профиль  
                  
 
 
Сообщение16.02.2007, 09:48 
Заслуженный участник
Аватара пользователя


07/03/06
1898
Москва
1) Немного переиначим. Нужно найти бесконечно много $m$, что
$\sum\limits_{i=1}^{\infty}{\lfloor \frac{2m}{p^i}} \rfloor-2\sum\limits_{i=1}^{\infty}{\lfloor \frac{m}{p^i}} \rfloor=0$ одновременно для $p=3,5,7$.
Отдельно рассмотрим это выражение для $p=3,5,7$.
$p=3$, тогда это выражение выполняется, если $m=3^k$ или $m=3^k+1$ (не все, но только очевидные случаи)
$p=5$, тогда это выражение выполняется, если $m=5^k$ или $m=5^k+1$ или $m=5^k+2$ (не все, но только очевидные случаи)
$p=7$, тогда это выражение выполняется, если $m=7^k$ или $m=7^k+1$ или $m=7^k+2$ или $m=7^k+3$ (не все, но только очевидные случаи).
Можно посмотреть при каких $k$ это будет выполняться одновременно, например, $m=1,10$ - выполняется.
Это, конечно, не решение, а мысли вслух.
Добавлено
Очевидные случаи не помогают, т.к. уравнения типа $A^x-B^y=C$ имеют, как правило, конечное число решений.

 Профиль  
                  
 
 
Сообщение16.02.2007, 11:25 
Заслуженный участник


09/02/06
4382
Москва
С точки зрения вероятностных соображений утверждения справедливы. Думаю, что авторы воспользовались именно этими соображениями. Встречал я эти задачи вссвязи с числами Каталана. Кажется даже здесь обсуждали.
Вероятностные соображения следующие. Количество чисел меньше большого числа N имеет все цифры в p -ичном исчислении меньше p/2 с вероятностью $(\frac{p+1}{2p})^{N_p}, \ N_p=\frac{log(N)}{log p}$. Поэтому, по вероятностным соображениям найдётся бесконечно много натуральных чисел n, что все цифры меньше p/2 для всех нечётных p из некоторого множества тогда и только тогда, когда $\sum_p\frac{\ln(2p)-\ln(p+1)}{\ln(p)}<1$. Всё это верно для множества из p=3,5,7 (сумма равна 0.974...) и для множества p=3,5,7,11 сумма превосходит 1 (примерно 1.2268...).

 Профиль  
                  
 
 
Сообщение16.02.2007, 11:33 
Заслуженный участник
Аватара пользователя


07/03/06
1898
Москва
Расчеты показывают, что для п.1 из двух миллионов имеем только такие решения:
1
10
756
757
3160
3186
3187
3250
7560
7561
7651
20007

 Профиль  
                  
 
 
Сообщение16.02.2007, 14:15 
Заслуженный участник


09/02/06
4382
Москва
По вероятностным соображениям n -ое такое число порядка n в степени 38 (0,974/(1-0.974)). Так, что в дальнейшем их должно встретится реже. Вначале, когда количество цифр мало, это немного нарушается и встречается несколько чаще.

 Профиль  
                  
 
 
Сообщение17.02.2007, 07:46 
Заслуженный участник
Аватара пользователя


07/03/06
1898
Москва
Мне кажется, что найти бесконечность можно уже среди чисел специального вида - $m=2^{k_1}\cdot 3^{k_2} \cdot 5^{k_3} \cdot 7^{k_4}$.

 Профиль  
                  
 
 
Сообщение01.03.2007, 15:33 


23/01/07
3419
Новосибирск
Допустим, что число m в 3-чной системе счисления (СС) имеет вид:
$ A = ...a_6 a_5 a_4 a_3 a_2 a_1 a_0 $,
где
$ a_0 = 1$
$ a_1,a_2,a_3,a_4,a_5,a_6,... < 2 $

Мне кажется, что решение этих задач "кроется" в следующих системах неравенств:

$ (a_0*1 + a_1*3 + a_2*4 + a_3*2 + a_4*1 + a_5*3 + a_6*4 + ... )\equiv (< 3)(mod 5) $ (1)
$ (a_0*1 + a_1*3 + a_2*2 + a_3*6 + a_4*4 + a_5*5 + a_1*1 + ...) \equiv (< 4)(mod 7) $


$ (a_0*1 + a_1*3 + a_2*4 + a_3*2 + a_4*1 + a_5*3 + a_6*4 + ... )  \equiv (< 3)(mod 5) $ (2)
$ (a_0*1 + a_1*3 + a_2*2 + a_3*6 + a_4*4 + a_5*5 + a_6*1 +.... ) \equiv (< 4)(mod 7) $
$ (a_0*1 + a_1*3 + a_2*9 + a_3*5 + a_4*4 + a_5*1 + a_6*3 +...)  \equiv (< 5)(mod 11)$

Суть по-видимому, в том, что система неравенств (1) выполняется бесконечно, а система неравенств (2) имеет ограничение по величине числа m.

 Профиль  
                  
 
 
Сообщение03.03.2007, 10:53 


23/01/07
3419
Новосибирск
Допустим, что число m в 1155-чной системе счисления (СС) имеет вид:
$ A = ...a_6 a_5 a_4 a_3 a_2 a_1 a_0 $,
где $ a_0 $ не кратно 3, 5, 7,11

Тогда для 2-й задачи можно записать:

$ (a_0*1 + a_1*0 + a_2*0 + a_3*0 + a_4*0 + a_5*0 + a_6*0 + ... )\equiv (< 2)(mod 3) $ (2)
$ (a_0*1 + a_1*0 + a_2*0 + a_3*0 + a_4*0 + a_5*0 + a_6*0 + ... )\equiv (< 3)(mod 5) $
$ (a_0*1 + a_1*0 + a_2*0 + a_3*0 + a_4*0 + a_5*0 + a_6*0 + ... )\equiv (< 4)(mod 7) $
$ (a_0*1 + a_1*0  + a_2*0 + a_3*0 + a_4*0 + a_5*0 + a_6*0 +...)\equiv (< 5)(mod 11)$

Хотя эти условия и не достаточные, но возникает мысль о том, что может быть, достаточно проверить, имеет ли 2-я задача решение в пределах $ 1< m < 1155 $?

 Профиль  
                  
 
 
Сообщение01.03.2009, 09:27 
Модератор
Аватара пользователя


11/01/06
5660
juna в сообщении #54025 писал(а):
Расчеты показывают, что для п.1 из двух миллионов имеем только такие решения:

см. последовательность A030979

 Профиль  
                  
 
 Re: две 1000-долларовые задачи от Рона Грэхема
Сообщение21.10.2010, 03:44 
Заслуженный участник
Аватара пользователя


11/01/06
3822
Здесь вроде как утверждается, что первая задача решена. Сам пока не смотрел.
Стиль изложения у автора какой-то совершенно отвратительный: много мусора, и невнятно местами.

 Профиль  
                  
 
 Re: две 1000-долларовые задачи от Рона Грэхема
Сообщение21.10.2010, 14:29 
Заслуженный участник


09/02/06
4382
Москва
RIP в сообщении #364298 писал(а):
Здесь вроде как утверждается, что первая задача решена. Сам пока не смотрел.
Стиль изложения у автора какой-то совершенно отвратительный: много мусора, и невнятно местами.

Бегло просмотрел. Там еще ссылки на предыдущие результаты. В принципе идея доказательства этой задачи понятная. Однако, эти методы не могут быть применены к другой задаче. Т.е. такими методами не удастся доказать конечность $(p_i,\frac{p_i-1}{2})$ good чисел для всех i, когда $\sum_i \frac{log (2p_i) -log (p_i+1)}{log p_i} >1.$

 Профиль  
                  
 
 Re: две 1000-долларовые задачи от Рона Грэхема
Сообщение22.10.2010, 02:09 
Модератор
Аватара пользователя


11/01/06
5660
RIP в сообщении #364298 писал(а):
Здесь вроде как утверждается, что первая задача решена. Сам пока не смотрел.

Тот же автор в другом препринте доказывает гипотезу Биля: http://arxiv.org/abs/1007.5122
Как-то подозрительно всё это...

 Профиль  
                  
Показать сообщения за:  Поле сортировки  
Начать новую тему Ответить на тему  [ Сообщений: 12 ] 

Модераторы: Модераторы Математики, Супермодераторы



Кто сейчас на конференции

Сейчас этот форум просматривают: нет зарегистрированных пользователей


Вы не можете начинать темы
Вы не можете отвечать на сообщения
Вы не можете редактировать свои сообщения
Вы не можете удалять свои сообщения
Вы не можете добавлять вложения

Найти:
Powered by phpBB © 2000, 2002, 2005, 2007 phpBB Group